Download as pdf
Download as pdf
You are on page 1of 135
PASTEST 2020 Respiratory AND ENT FOP PA AO a) Qbank 65 Questions successfully downloaded Ready to begin? 01:21 67% © eel ao} A 5-year-old boy has had multiple previous admissions for ‘virally induced wheeze’. He has been using a beta-agonist inhaler at home on an as-required basis. His mother tells you that he becomes wheezy with changes in temperature, laughter, exercise and with viral illnesses. He must use his inhaler three times per week and coughs on most days, usually at night. What is the next best step in his management? Select one answer only Increase the number of puffs of his beta-agonist inhaler A B| Add a low-dose steroid inhaler Cc D EB Add a high-dose steroid inhaler Add a long-acting beta-agonist Add ipratropium inhaler rc aR a) ‘A 5-year-old boy has had multiple previous admissions for ‘virally induced wheeze’. He has been using a beta-agonist inhaler at home on an as-required basis. His mother tells you that he becomes wheezy with changes in temperature, laughter, exercise and with viral illnesses, He must use his inhaler three times per week and coughs on most days, usually at night. What is the next best step in his management? Select one answer only Increase the number of puffs of his beta-agonist inhaler A c D E Add a high-dose steroid inhaler Add a long-acting beta-agonist ‘Add ipratropium inhaler Explanation This child has a history of reversible airway obstruction associated with viral infections (virally induced wheeze’) but is now presenting with episodic symptoms from other triggers, such as exercise and laughter. As such, he has a high probability of asthma, and he should be started on a low-dose inhaled corticosteroid therapy for prevention. This should be coupled with revision of inhaler technique and subsequent review of symptoms after six weeks or so. A good response to treatment confirms the diagnosis. DO ea Beta-agonists provide symptomatic relief in childhood asthma, but will not halt the progression of disease, which is already poorly controlled. Preventer therapy is indicated OCC Inhaled corticosteroids should be started at a very low dose in children and titrated initially up to the lowest dose at which effective control of asthma is maintained, Deere cca cad A long-acting beta-agonist is indicated as an initial add-on therapy in children whose asthma is poorly controlled with an inhaled corticosteroid and short-acting beta-agonist. Ipratropium is used as a bronchodilator therapy in acute asthma attacks in children. Itis not. routinely used as a reliever therapy outside this setting, CORTE MStoLr eal Question 2 of 65 Theme: asthma management A Salbutamol inhaler regularly for the next 12 hours and three times in the first hour B Short-term B agonist inhaler via spacer C Salbutamol nebulisers regularly for the next 12 hours and three times in the first hour D Start ipratropium via appropriate inhaler regularly E Start ipratropium via appropriate inhaler as required F Give a 5 day course of oral steroids G Give steroid via appropriate inhaler regularly H Give steroid via appropriate inhaler as required I No treatment required For each of the scenarios, choose the first ‘short term’ plan of action. Each option may be used once, more than once, or not at all. ‘A3-year-old boy presents with episodic wheeze worse when playing or running around. Please select > AT-year-old known asthmatic boy presents with a 2-hour history of acute shortness of breath. He has widespread wheeze but is talking and his 02 saturations are 94% in room air. Please select > ‘10-year-old known asthmatic attends surgery with a painful right toe. He has had several hospital admissions for asthma in the past, but has not had an asthma attack for one year. He has no night-time symptoms and describes no exercise limitation. He was prescribed daily inhaled steroids via large volume spacer 4 years ago, but his mother concedes that he has not been using this much over the last 12 months. Please select > er Cnr) Theme: asthma management A Salbutamol inhaler regularly for the next 12 hours and three times in the first hour B Short-term B agonist inhaler via spacer C Salbutamol nebulisers regularly for the next 12 hours and three times in the first hour D Start ipratropium via appropriate inhaler regularly E Start ipratropium via appropriate inhaler as required F Give a5 day course of oral steroids G Give steroid via appropriate inhaler regularly H Give steroid via appropriate inhaler as required INo treatment required For each of the scenarios, choose the first ‘short term’ plan of action. Each option may be used once, more than once, or not at all. ‘A 3year-old boy pretents with episodic wheeze worse when playing or running around 8. Short-term B agonist inhaler via spacer S Explanation This child is experiencing symptoms of acute asthma triggered by exercise. In this instance a short-acting 8 agonist via spacer should be the first line treatment. Therepy should be reviewed at a later date to assess the frequency of use and adequacy of symptom control. 'A 7-year-old known asthmatic boy presents with a 2-hour history of acute shortness of breath. He has widespread wheeze but is taking and his 02 saturations are 94% in room a. ‘A, Salbutamol inhaler regularly for the next 12 hours and 3 times in the 1st hour aS Explanation The 7-year-old boy is having a moderate acute exacerbation of asthma. As he is not requiring ‘oxygen, inhalers are the most appropriate treatment. He would benefit from having 10 puffs every 20 minutes for the first hour and will then need to use the inhaler regularly for the next 12 hours ‘and possibly even longer. ‘Therapy should be reviewed according to clinical response and a short course of steroids considered if symptoms have not resolved quickly 10-year-old known asthmatic attends surgery with a painful right toe. He has had several hospital admissions for ‘asthma in the past, but hae not had an asthma attack for one year. He has no night-time symptoms and describes no exercise limitation. He was prescribed dally inhaled steroids via large volume spacer 4 years ago, but his mother concedes that he has not been using this much over the last 12 months. Lenn aS Explanation The 10-year-old boy has no interval symptoms despite not using his inhaled steroids. it would be appropriate to stop all his treatment if he has not required inhaled steroids and has been symptom free for one year. Asthma therapy should be reviewed regularly. Rate this Tag Question zt) sl-led Question 3 of 65 A 4-year-old girl is brought to the General Practitioner with a 3-day history of fever, coryza and productive cough. On examination, she is mildly tachypnoeic, with a respiratory rate of 31 breaths/min but an oxygen saturation of 96%. There are subcostal recessions, and there are coarse end-expiratory crackles in both lung bases on auscultation. She is previously well and has not had any hospital admissions. What is the most likely underlying pathogen? Select one answer only Streptococcus pneumoniae Mycoplasma pneumoniae Staphylococcus aureus Haemophilus influenzae m)/oUl/oO;|oD|> Viral pneumonia er C nn) ‘A 4-year-old girl is brought to the General Practitioner with a 3-day history of fever, coryza and productive cough. On examination, she is mildly tachypnoeic, with a respiratory rate of 31 breaths/min but an oxygen saturation of 96%. There are subcostal recessions, and there are coarse end-expiratory crackles in both lung bases on auscultation. She is previously well and has not had any hospital admissions. What is the most likely underlying pathogen? Select one answer only ‘Streptococcus pneumoniae Mycoplasma pneumoniae A B C | staphylococcus aureus D Haemophilus influenzae Explanation x The clinical picture is consistent with a diagnosis of community-acquired pneumonia, and in this ‘age group, viruses are the most common cause (eg respiratory syncytial virus or rhinovirus). This is further suggested by the presence of coryza, along with the bilateral chest findings. Bacterial infections are also common, however, and mixed viral and bacterial infections do occur. As such, antibiotic cover is usually recommended. ‘S. pneumoniae is a common cause of pneumonia in infants and young children, but is typically associated with more localised signs and symptoms and viruses are more common in this age group, M. pneumoniae is associated with a diffuse picture, as seen here, but is rare in children of this, age. S. aureus does not commonly cause pneumonia in children, unless there is a particular predisposition, for example, in cystic fibrosis. H. influenzae is an important cause of pneumonia in young children, but routine vaccination offers ‘a great deal of protection and viral pneumonia is more common, CoRUIen MS toUr — Question 4 of 65 A 16-year-old trainee nursery nurse presents to A&E acutely unwell. She has a cough and has become progressively worse, now with severe shortness of breath. Her flatmate reports that she has become unsteady on her feet over the past few days. There is no important past history of note, she smokes 20 cigarettes per day. There is a striking rash over her scalp, face and trunk with a mixed crop of vesicles and pustules suggestive of chickenpox. On examination she has bilateral cerebellar ataxia. Which diagnosis fits best with this clincal picture? Acute varicella zoster virus infection with CNS and lung involvement Bacterial pneumonia Herpes simplex encephalitis Generalised viral infection of unknown origin m)/OU/O;o|> Likely inmunocompromised patient Question 4 of 65 A 16-year-old trainee nursery nurse presents to A&E acutely unwell. She has a cough and has become progressively worse, now with severe shortness of breath. Her flatmate reports that she has become unsteady on her feet over the past few days. There is no important past history of note, she smokes 20 cigarettes per day. There is a striking rash over her scalp, face and trunk with a mixed crop of vesicles and pustules suggestive of chickenpox. On examination she has bilateral cerebellar ataxia. Which diagnosis fits best with this clincal picture? Acute varicella zoster virus infection with CNS and lung involvement aS B c ephalitis D viral infection of unknown origin E | Likely inmunocompromised patient Explanation Although uncommon, lung involvement with acute chickenpox infection is seen. The pneumonitis generally begins some 1-6 days after the appearance of the skin eruption and is more common in young adults, with cigarette smokers being particularly at risk. This woman was not exposed to chickenpox as a child, and exposure has now occurred due to her work as a nursery nurse. The chest radiograph normally shows diffuse lung changes throughout both lung fields, but symptoms are commonly worse than the X-ray picture. CNS involvement is rarer, occurring in around 1 in 1000 cases, and presenting with acute truncal ataxia. Treatment is with intravenous aciclovir, with support from the high- dependency/intensive care unit as needed. el CM Me [Plast Tag Question Feedback Question 5 of 65 A 16-year-old girl presents to the Emergency Department with an exacerbation of her asthma On examination, her observations are respiratory rate 32 breaths/min, oxygen saturation 91%, blood pressure 123/80 mmHg, heart rate 110 bpm and temperature 36.5 °C. There is widespread wheeze on auscultation of the chest. She is able to talk, but is taking two or three breaths to complete a sentence. What is the most important therapy to relieve her bronchoconstriction? Select one answer only Propranolol Salbutamol Oxygen Glucocorticosteroids m)/oUl/oO;|oD|> Cromoglicate ern) ‘A 16year-old girl presents to the Emergency Department with an exacerbation of her asthma On examination, her observations are respiratory rate 32 breaths/min, oxygen saturation 91%, blood pressure 123/80 mmHg, heart rate 110 bpm and temperature 36.5 °C. There is widespread wheeze on auscultation of the chest. She is able to talk, but is taking two or three breaths to complete a sentence. What is the most important therapy to relieve her bronchoconstriction? Select one answer only Propranolol A Cc D E Oxygen Glucocorticosteroids Cromoglicate Explanation Salbutamol is a beta-agonist that acts selectively on beta-2-adrenoceptors on bronchial smooth muscle to cause bronchodilation. It also inhibits mast-cell degranulation. In a severe attack such as this, it should be administered by nebulised solution, in combination with oxygen therapy titrated to saturations of 94-98%. Corticosteroids should be given early, orally if tolerated, and additional bronchodilator therapies may be indicated if her condition does not rapidly improve. Propranolol is a non-selective beta-blocker, used for thyrotoxicosis and migraine prophylaxis. It ‘would likely cause worsening bronchoconstriction, which could be life-threatening, Oxygen is essential to treat this girs hypoxia, but it will not reverse the bronchoconstriction. DMM t Oral corticosteroids should be given early to reduce airway inflammation and reverse bronchoconstriction, but it wll be hours before they take effect. This prevents release of mast-cell mediators and is used for prophylaxis of severe attacks in children over the age of five, in whom inhaled corticosteroids are contraindicated. Itis not useful inacute asthma attacks — Question 6 of 65 12-year-old girl is admitted to the Emergency Department. She had been found by her mother and younger sister when they returned back from shopping. When they got back to the flat, they tell you she was unsteady on her feet and having problems with her memory. They tell you they have been using old fashioned gas fires as it has been so cold. You are wondering whether she could have carbon monoxide poisoning. Which test will be most helpful in determining this? Clinical examination Pulse oximetry Arterial blood oxygen level Exhaled breath test m)/OU)/O/a/]> Chest radiograph Question 6 of 65 12-year-old girl is admitted to the Emergency Department. She had been found by her mother and younger sister when they returned back from shopping. When they got back to the flat, they tell you she was unsteady on her feet and having problems with her memory. They tell you they have been using old fashioned gas fires as it has been so cold. You are wondering whether she could have carbon monoxide poisoning. Which test will be most helpful in determining this? Clinical examination Arterial blood ox D| Exhaled breath test S Chest radiograph A B ce yen level Explanation Breath carbon monoxide levels can be measured using devices intended for smoking cessation work. They can also be measured on a co-oximeter, measuring carboxyhaemoglobin levels. Clinical examination is unhelpful, as patients are not cyanosed, but a cherry-red colour. Pulse oximetry appears normal, due to carboxyhaemoglobin having similar absorption spectra to oxyhaemoglobin. Arterial oxygen levels may be normal, as may a chest radiograph. Treatment for carbon monoxide poisoning is to give high levels of inspired oxygen, by ventilation (or hyperbaric means if necessary), to displace the carbon monoxide and increase the amount of dissolved oxygen el CM Me [Plast Tag Question Feedback Question 7 of 65 ‘A 2-year-old girl presents to your Emergency Department with a history of cough and a 6-hour history of noisy breathing. She has a barking cough, mild recession and mild stridor. What is the best course of action? Take her to the resuscitation area and ask anaesthetists to attend urgently Give an oral dose of dexamethosone (600 micrograms/kg) Request a lateral X-ray Give an oral dose of dexamethasone (150 micrograms/kg) and observe for a further 4 hours m/oU/O;|o|> Prescribe oral budesonide 1mg/kg Question 7 of 65 ‘A 2-year-old girl presents to your Emergency Department with a history of cough and a 6-hour history of noisy breathing. She has a barking cough, mild recession and mild stridor. What is the best course of action? B | Give an oral dose of dexamethosone (600 micrograms/kg) ral X-ray observe for a further 4 hours G cy Give an oral dose of dexamethasone (150 micrograms/kg) and aS E Prescribe oral budesonide 1mg/kg Explanation The child has croup and she is showing signs of mild illness. In an A&E department, the most appropriate thing to do would be to give an oral dose of dexamethasone (150 micrograms/kg) and observe for a further 4 hours. There is little evidence for the higher dose strategy giving more benefits than the lower one. An X-ray would not help distinguish the diagnosis and as she is maintaining her own airway there is no need for an anaesthetist. EVM eee (Lat Tag Question Feedback Question 8 of 65 A7-year-old girl presents to the Emergency Department with a high fever and a painful cough productive of purulent sputum. A right lower-lobe pneumonia is confirmed by X-ray. Nine months previously, she was hospitalised for three days with similar symptoms, but made a full recovery. The chest X-ray report described similar right lower-lobe changes. There is no contact history of tuberculosis. She has not had the Bacillus Calmette—Guérin (BCG) vaccine, but the rest of her immunisation history is up to date What is the next best step in her management? Select one answer only Bronchoscopy Mantoux test Computed tomography (CT) scan of the thorax Repeat chest X-ray with clinical review in six weeks m)/OU;/O/;/oO/]> Ventilation perfusion scan er C na) ‘A 7-year-old girl presents to the Emergency Department with a high fever and a painful cough productive of purulent sputum. A right lower-lobe pneumonia is confirmed by X-ray. Nine months previously, she was hospitalised for three days with similar symptoms, but made a full recovery, The chest X-ray report described similar right lower-lobe changes. There Is no contact history of tuberculosis. She has not had the Bacillus Calmette~Guerin (BCG) vaccine, but the rest of her immunisation history is up to date What is the next best step in her management? Select one answer only Bronchoscopy Mantoux test A B C | Computed tomography (CT) scan of the thorax E Ventilation perfusion scan Explanation This child has presented with two respiratory infections affecting the same area of the lung within a relatively short space of time. This may be due to a foreign body, a chronic infection like TB or ‘an immunodeficiency, but given that she recovered well after the first one and has no other ‘medical history, itis most likely to be coincidental. The most sensible course would be to monitor for resolution of the infection with repeat imaging and a clinical review. Bronchoscopy may be indicated to exclude a foreign body if symptoms do not improve, but the history of good recovery from the initial infection makes this unlikely. aura cry The risk of tuberculosis (TB) is low, given that she recovered well from the first infection. As such, it would be worth monitoring for clinical and radiological improvement before conducting further investigations. €. Computed tomography (CT) scan of the thorax ACT scans very sensitive for chronic lung conditions that predispose to infection, like bronchiectasis, but it would be a high dose of radiation for a young child and should only be considered if symptoms do not resolve. Pu Cy Ventilation perfusion scan may show evidence of poor ventilation in an area of consolidation, but this information is already available from the chest Xray. Chuo teeter _ Question 9 of 65 Theme: Respiratory medicine A Intensive chest physiotherapy B 24-hour oesophageal pH monitoring C Prescribe low dose inhaled steroids via a spacer +/- a mask D Measure sweat electrolytes E Request a diary of twice daily peak flow measurements F Refer to the dietician for a detailed nutritional assessment G Prescribe 2 weeks of oral ciprofloxacin H Oral amoxicillin for a 7 day course | Refer for a routine flexible bronchoscopy For each of the scenarios below, select the most appropriate way to proceed from the options above. Each option may be used once, more than once, or not at all. ‘A2year-old child with poor growth has seen the GP with recurrent chestiness and delayed speech. He is on inhaled sodium cromoglycate via a spacer with equivocal benefit. He was admitted at 9 months of age with bronchiolitis. In a recent referral to the ENT department, nasal polyps were detected and he was prescribed intranasal steroids. Please select > ‘A 4-year-old girl has had troublesome nocturmal cough on about 4 nights per week for over a year. Another GP thought she might have asthma and prescribed a salbutamol inhaler which seems to have improved symptoms slightly, Please select > ‘A.23-month-old boy presents to you with mild breathlessness, fever and cough for just under 24 hours. His parents report that he is a very active boy who is always running around and is ‘into everything’. On examination he has decreased air entry on the right with occasional wheeze. Please select > ere Ca) Theme: Respiratory me A Intensive chest physiotherapy B 24-hour oesophageal pH monitoring C Prescribe low dose inhaled steroids via a spacer +/- a mask D Measure sweat electrolytes E Request a diary of twice daily peak flow measurements F Refer to the dietician for a detailed nutritional assessment G Prescribe 2 weeks of oral ciprofloxacin H Oral amoxicillin for a 7 day course | Refer for a routine flexible bronchoscopy For each of the scenarios below, select the most appropriate way to proceed from the options. above. Each option may be used once, more than once, or not at all. Eon ‘A 2year-old child with poor growth has seen the GP with recurrent chestiness and delayed speech. He is on inhaled sodium cromoglycate via a spacer with equivocal benefit. He was admitted at 9 months of age with bronchiolitis. In a recent referral tothe ENT department, nasal polyps were detected and he was prescribed intranasal steroids, ine Leet tonnline ao Explanation This child is displaying features suggestive of CF, in which case a sweat testis the most ‘appropriate thing to do next from the options given. ‘A 4yyear-old gir has had troublesome nocturnal cough on about 4 nights per week for over a year. Another GP. ‘thought che might have asthma and preceribed a ealbutamol Inhaler Which ceems to have improved symptome stightly C. Prescribe low dose inhaled steroids vi ‘spacer +/- mask S Explanation This child appears to have asthma and needs to be treated with low dose inhaled steroids via a spacer to ensure correct administration of the medication in a four year old. Inhaled steroids are effective in reducing inflammation and preventing nocturnal symptoms. ‘A 23:monttrold boy presents to you with mild breathlessness, fever and cough for just under 24 hours. His parents report that he is very active boy who is always running around and is'into everything. On examination he has ‘decreased air entry on the right wth occasional wheeze. Hel amoxil for 7 day course oS Explanation This child has a right sided pneumonia and needs oral antibiotics for 7 days. Foreign body is a differential, but there is no history of a choking episode to suggest bronchoscopy may be indicated. If foreign body was suspected to be the cause of the pneumonia, an urgent bronchoscopy (not routine) would be needed. Tag Question ora Question 10 of 65 Theme: Respiratory conditions Aa-1 anti-trypsin B Atopic asthma C Cystic fibrosis D Foreign body inhalation E Kartagener syndrome F Mycoplasma infection G Pertussis H Recurrent aspiration | Vascular ring J Tuberculosis For each of the following cases please choose the most likely diagnosis from the above list. Each item may be used once, more than once or not at all. A 2-year-old is referred with difficulty in breathing, She is noted to be wheezy ~ more on the right than left. A chest X-ray reveals right-sided hyperexpansion. Paes cls > An 18-month-old infant of a travelling family presents with an acute respiratory illness. Investigations at the time show 2 marked lymphocytosis. He has a persistent cough 2 months later. Please select > A3-year-old child has a recurrent noctumal cough. He had mild eczema as an infant. He is otherwise well Please select > Or um ey Theme: Respiratory conditions Aa- antitrypsin B Atopic asthma C Cystic fibrosis D Foreign body inhalation E Kartagener syndrome F Mycoplasma infection G Pertussis H Recurrent aspiration | Vascular ring J Tuberculosis For each of the following cases please choose the most likely diagnosis from the above list. Each item may be used once, more than once or not at all. ‘A 2year-old is referred with dificult in breathing. She is noted to be wheezy ~ more on the right than left. A chest X-ray reveals right-sided hyperexpansion . Foreign body inhalation oS Explanation Ina toddler who presents with difficulty in breathing, foreign body aspiration must always be considered. Toddlers are inquisitive and will put most small objects in their mouths. Any ‘asymmetry in expansion or wheeze should increase the index of suspicion, ‘An 18-month-old infant ofa traveling family presents with an acute respiratory illness. Investigations atthe time chow a marked lymphocytosis. He has a persistent cough 2 months later. opi ao Explanation ‘An immunisation history must always be asked for (and the personal child health record (red book’) should be used). A cough that occurs in paroxysms, subconjunctival haemorrhages from repeated coughing and a marked lymphocytosis are all classic features of whooping cough Treatment is supportive but azithromycin can be given to limit infectivity to other children. The ‘cough can last up to 3 months (the 100 days cough). ‘A Byyear-old child has a recurrent nocturnal cough. He had mild eczema as an infant. He is otherwise well B. Atopic asthma S Explanation A previous history of an atopic condition or a family history of atopy increases the risk of asthma (and other atopic conditions). A nocturnal cough is a recognised presentation of asthma in pre- school children. A trial of an inhaled bronchodilator via a spacer is worthwhile Explanation Tag Question ioe a Question 11 of 65 A9-year-old girl with known asthma is brought to the General Practitioner with a history of worsening cough at night, as well as increasing inhaler use during the day. She is using her inhaler up to three times a day, and on most days. She is currently on a low-dose inhaled corticosteroid once a day, as well as a salbutamol inhaler PRN (as required). On examination, her observations are normal and there is no wheeze. What is the most appropriate next step in management? Select one answer only Add long-acting beta-2-agonist Increase the dose of inhaled steroid Check the inhaler technique Add a leukotriene inhibitor m)/oUl/oO;|oD|> Add a short course of oral steroids Cr UR) ‘A 9-year-old girl with known asthma is brought to the General Practitioner with a history of worsening cough at night, as well as increasing inhaler use during the day. She is using her inhaler up to three times @ day, and on most days. She is currently on a low-dose inhaled corticosteroid once a day, as well as a salbutamol inhaler PRN (as requited). On examination, her observations are normal and there is no wheeze. What is the most appropriate next step in management? Select one answer only Add long-acting beta-2-agonist Increase the dose of inhaled steroid A B D_ | Add a leukotriene inhibitor E fo) Add a short course of oral steroids Explanation % In managing patients with poorly controlled asthma, the priorities are to check adherence to current treatment, check inhaler technique and eliminate trigger factors through careful history- taking before considering any changes to medication. In young children, a pressurised metered dose inhaler plus spacer is the preferred method of delivery, to ensure optimal distribution, Pere eee This could be considered as an initial add-on therapy in a child whose asthma is poorly controlled with inhaled corticosteroid, but the priority is to ensure correct inhaler technique. cnr Peg cl) This is usually only done in children who are already being treated with an add-on therapy such as a long-acting beta-agonist or leukotriene-receptor antago Deu og ‘Adding a leukotriene inhibitor could also be considered as an initial add-on therapy in a child whose asthma is not well controlled; the first priority is to check the inhaler technique. Pee CC The child is currentiy well, and in a primary care setting, oral steroids are only prescribed for acute ‘exacerbations of asthma Pcnae _ on Question 12 of 65 12-year-old girl is diagnosed as having a generalised chest infection, after presenting with a 10-day history of gradually worsening cough, which is now productive. Her respiratory rate is 22 breaths/min and oxygen saturation 98% in air; there is no recession. The differential includes mycoplasma as a causative organism. What would be the most appropriate course of action? Select one answer only Admit for intravenous (IV) antibiotics Allow home but arrange for IV antibiotics to be given by the homecare nurses A 7-day course of oral amoxicillin A 7-day course of oral erythromycin m!/OU/O/]/o/] > Admit for oral antibiotics and observation Or Une) ‘A 12-year-old girl is diagnosed as having a generalised chest infection, after presenting with a 10-day history of gradually worsening cough, which is now productive. Her respiratory rate is 22.breaths/min and oxygen saturation 98% in air, there is no recession. The differential includes mycoplasma as a causative organism. What would be the most appropriate course of action? Select one answer only Admit for intravenous (IV) antibiotics ‘Allow home but arrange for lV antibiotics to be given by the homecare nurses A 7-day course of oral amoxicillin A B c E Admit for oral antibiotics and observation Explanation ® The generalised features and slow progression of symptoms are suggestive of Mycoplasma pneumoniae, which is a common cause of pneumonia in children above the age of five Mycoplasma infections generally respond well to a macrolide or to doxycycline but not amoxicillin. COS om neuen) Euan iecy This would only be indicated if there were evidence of sepsis or a complication such as ‘empyema. Most pneumonias respond well to oral antibiotics. EUS Pee gc There is no indication for lV antibiotics, given that most chest infections respond well to oral treatment. COCA ee eo) This would generally be first line for community-acquired pneumonia in children, but amoxicillin is often ineffective against mycoplasma, which is the most likely causative agent in context of the ‘generalised features eMC Lc Most chest infections in children can be managed in the community. Indications for hospital admission include hypoxia, severe tachypnoea, respiratory distress or signs of sepsis, none of which are present here. CORTE MStoLr eal Question 13 of 65 ‘A 5-year-old boy has a history of recurrent nocturnal cough and frequent difficulty in breathing on exercise. He has recently been admitted with moderately severe wheeze that responded to nebulised bronchodilator therapy and a dose of oral prednisolone. What is the best management after discharge? Select one answer only A home nebuliser with salbutamol as needed Becotide 200 pg twice daily via a metered dose inhaler (MDI) Fluticasone propionate (Flixotide®) 100 1g twice daily via a low- volume spacer Budesonide 200 ug twice daily via a dry powder device m)/O0;/O;wao]|> Montelukast 5 mg at night Or URE) ‘A 5.year-old boy has 2 history of recurrent nocturnal cough and frequent difficulty in breathing on exercise. He has recently been admitted with moderately severe wheeze that responded to nebulised bronchodilator therapy and a dose of oral prednisolone. What is the best management after discharge? Select one answer only, Ahome nebuliser with salbutamol as needed Becotide 200 jig twice dally via a metered dose inhaler (MDI) Budesonide 200 ig twice daily via a dry powder device mo Ryo > Montelukast 5 mg at night Explanation SUR eae er Tat The history of recurrent episodic wheeze, as well as nocturnal coughing is suggestive of childhood asthma. This should be managed intially with a trial of low-dose inhaled corticosteroids. If his symptoms improve, this would confirm the diagnosis. Administration should be by a spacer for a child of this age, in order to ensure adequate delivery into the lungs ‘and minimise risk of oral candidiasis PO merece) Home nebulisers are not routinely used in the management of childhood asthma. This child needs a tral of preventer therapy, which should be a low-dose inhaled corticosteroid aC Oe oe Moe eu a) This would be a suitable choice of medication as an initial preventer therapy, but it should be administered with a spacer in @ child of this age. Dee eet RET oe cr ce ace This is not an appropriate delivery method for a child of this age, particularly for a steroid inhaler. Deeg Leukotriene antagonists are not a first-line therapy in asthma prevention. They are indicated as an add-on therapy in children who are already using a regular inhaled corticosteroid. _ Question 14 of 65 A 4-year-old child has a 2-day history of fever and also presents with a bright-red, infected, left tympanic membrane. The child is eating and drinking well. What would be the most appropriate course of action? Referral to ENT (ear, nose, throat) surgeons as an emergency Referral to ENT surgeon as an outpatient 5-day course of oral phenoxymethylpenicillin Advice to parents that this is a self-limiting condition and is best left alone m)/oOU/O/;|o|> Regular antipyretics and analgesics with GP review in 48h Or URC) ‘A 4-year-old child has a 2-day history of fever and also presents with a bright-red, infected, left tympanic membrane. The child is eating and drinking well What would be the most appropriate course of action? Referral to ENT (ear, nose, throat) surgeons an emergency Referral to ENT surgeon as an outpatient of oral phenoxymethylpenicillin t left \dvice to parents that this is a self-limiting condition and is b alone A B Cc D | Regular antipyretics and analgesics with GP review in 48h ov Explanation ‘Antibiotics are of limited benefit in the treatment of otitis media, They can shorten the duration of the illness but the number needed to treat is 17. Of the cases of otitis media, 80% resolve without treatment. Supportive measures such as antipyretics and analgesics are important measures to recommend to parents. (Sharon Sanders, Paul P Glasziou, Chris Del Mar, and Maroeska Rovers. (2009) ‘Antibiotics for acute otitis media in children (Review)" Cochrane database of systematic reviews, (2), 1-43) NICE recommends that antibiotics are given immediately only to the following groups: + Systemically unwell but do not require admission, ‘= Athigh risk of serious complications because of significant heart, lung, kidney, liver, or neuromuscular disease/ immunocompromised. « Symptoms have lasted for four days or more and are not improving, Depending on severity, consider offering an immediate antibiotic prescription to children ‘+ Younger than two years of age with bilateral AOM. + With perforation and/or discharge in the ear canal It would be appropriate to escalate ifthe child!s symptoms worsen, or if there is no improvement within 4 days of the onset of symptoms Antibiotic treatment, if required, would be a 5 day course of amoxicillin een Tag Question Cle) el-loLd Question 15 of 65 ‘A 4-year-old boy is brought to the General Practitioner surgery two weeks after undergoing bilateral grommet insertion to treat chronic otitis media with effusion. His mother would like advice about preventing future infections. What is the most appropriate advice to prevent further middle ear infections? Select one answer only Swimming in a chlorinated swimming pool should be avoided Swimming in the sea should be avoided Earplugs should be worn during wet weather Aeroplane flights should be avoided m)/O0;/O;wao]|> Plugging the ears for baths and showers is advisable Or Rey ‘A 4-year-old boy is brought to the General Practitioner surgery two weeks after undergoing bilateral grommet insertion to treat chronic otitis media with effusion. His mother would like advice about preventing future infections. What is the most appropriate advice to prevent further middle ear infections? Select one answer only, Swimming in a chlorinated swimming pool should be avoided Swimming in the sea should be avoided Earplugs should be worn during wet weather A B c D Explanation Aeroplane flights should be avoided Itis important to keep the ears dry to prevent infections because the grommet potentially allows water to leak into the middle ear. The most likely time when this might occur is during dally washing. Soaps and shampoos are iritant to the middle ear, which confer additional risk. As such, itis advised that children use waterproof ear plugs when washing to protect the middle ear from getting wet. a eet EC ee ee ) ‘Swimming in chlorinated pools does confer a risk of infection but this can be safely mitigated by plugging the ears. Diving and underwater swimming should be discouraged Loe ec e nd ‘Swimming in the sea is safe, as long as earplugs are used and diving is avoided. Cae Oe DO Rec Itis important to keep the ears dry to prevent infections, but rainfall confers minimal risk and earplug use is not recommended. D. Aeroplane flights should be avoided Grommets will prevent any pain associated with pressure changes on aeroplane flights. There is 1 particular risk of infection. _ Question 16 of 65 A 4-year-old girl presents with coryzal symptoms, cough, fever and abdominal pain. There is no dysuria, and urine dipstick is unremarkable. On examination, the pain is in the left upper quadrant but is not severe. Which is the most likely diagnosis? Select one answer only Urinary tract infection (UTI) Appendicitis Pyelonephritis Left basal chest infection Mesenteric adenitis m)/O0;/O;wao]|> Or un) ‘A 4-year-old girl presents with coryzal symptoms, cough, fever and abdominal pain. There is no dysuria, and urine dipstick is unremarkable. On examination, the pain is in the left upper quadrant but is not severe, Which is the most likely diagnosis? Select one answer only, Urinary tract infection (UT!) Appendicitis Pyelonephritis A B c E Mesenteric adenitis Explanation Pneumonia in children is often accompanied by localised pain in the chest, neck or abdomen. In this case, there is a clear history of chest symptoms, as well as upper quadrant abdominal pain, which is typical of a chest infection. Treatment is with oral antibiotics, unless there are signs of sepsis of respiratory distress, in which case hospital admission may be indicated. ‘A. Urinary tract infection (UTI) UTI would commonly present with dysuria, increased urinary frequency and lower abdominal pain, none of which is reported here. A negative urine dipstick all but excludes the diagnosis. The pain caused by appendicitis usually starts centrally before migrating to the right iliac fossa. Atypical presentations can occur, but a chest infection is more likely Pyelonephritis is more commonly associated with flank tenderness, rather than pain in the left upper quadrant, and is excluded by the normal urine dipstick. Mesenteric adenitis most commonly occurs in the right lower quadrant, where it can be mistaken for appendicitis; it is usually preceded by a sore throat. _ Question 17 of 65 A7-year-old child presents complaining of itchy eyes and nasal congestion. Her mother reports the symptoms tend to come and go. They are generally worse during the summer months. Examination reveals mild conjunctival injection but nil else of note. What is the BEST option for management? Cetirizine Chlorphenamine Intranasal antihistamine Intranasal corticosteroid m)/OU/O/;/o/> Intranasal decongestant Or UR) ‘A 7-year-old child presents complaining of itchy eyes and nasal congestion, Her mother reports the symptoms tend to come and go. They are generally worse during the summer months. Examination reveals mild conjunctival injection but nil else of note. What is the BEST option for management? Bod Chlorphenamine Intranasal antihistamine B c D | Intranasal corticosteroid ES Intranasal decongestant Explanation Cetirizine is a non-sedating antihistamine and should be considered first line management of allergic rhinitis in this case, NICE guidelines state an oral antihistamine should be prescribed first line for the management of allergic rhinitis in the following situations: ‘+ Conjunctivitis is also present ‘= Children aged 2 to 5 years of age + Patient preference for oral treatment. Chlorphenamine is incorrect. Oral antihistamines are indeed fist line treatment in this case however Chlorphenamine (Piriton) is a sedating antihistamine, This is likely to impair educational performance. A non-sedating antihistamine should be used. CAE id Intranasal antihistamine is incorrect. Intransal antihistamines are recommended first line in most ‘cases with the exception of the following situations; the presence of symptomatic conjunctivitis, children aged 2 to 5 years of age, patient preference for oral treatment. In this case the child is. ‘complaining of itchy eyes suggesting allergic conjunctivitis. In most other cases intranasal antihistamines (azelastine) should be prescribed first line, PACE een) Intranasal corticosteroid is incorrect. Intranasal corticosteroids are only recommended first line in ‘cases of persistent symptoms of allergic rhinitis where nasal blockage is the primary complaint, ‘or nasal polyps are present. ae ea cud Intranasal decongestant is incorrect. Although nasal decongestants are useful in the management of allergic rhinitis oral antihistamines would be first line in this case. Cee ono eae OP kOe Question 18 of 65 A-year-old child presents complaining of nasal congestion and discharge. Her mother reports the symptoms tend to be worse during the summer months. The child denies any discomfort or itchiness of her eyes and on examination there is no evidence of conjunctival injection What is the BEST option for management? Cetirizine Chlorpeniramine Intranasal antihistamine Intranasal corticosteroid m)/OU;/O;|D|> Intranasal decongestant Or URL) ‘A 7-year-old child presents complaining of nasal congestion and discharge. Her mother reports the symptoms tend to be worse during the summer months. The child denies any discomfort or itchiness of her eyes and on examination there is no evidence of conjunctival injection. What is the BEST option for management? Cetirizine Chlorpeniramine A B D | intranasal corticosteroid [?) Intranasal decongestant Explanation Intranasal antihistamine ~ Intranasal antihistamines are recommended first line in most cases with the exception of the following situations; the presence of symptomatic conjunctivitis, children aged 2 to 5 years of age, patient preference for oral treatment Cetirizine is incorrect. Oral antihistamines are first line treatment in children 2-5 years of age or if conjunctivitis is also present (which is not the case in this question), otherwise for intermittent allergic rhinitis intranasal antihistamines are first line. When oral antihistamines are used a non- sedating antinistamine, such as cetirizine, is a better choice than a sedating antihistamine such ‘as chlorpeniramine (Piriton), Chlorpeniramine is incorrect. Oral antihistamines are first line treatment in children 2-5 years of age or if conjunctivitis is also present (which is not the case in this question), otherwise for intermittent allergic rhinitis intranasal antihistamines are first line. Chlorpheniramine (Piriton) is a sedating antinistamine, likely to impact on educational performance, when oral antihistamines are used a non-sedating antihistamine is a better choice. Paci eer) Intranasal corticosteroid is incorrect. Intranasal corticosteroids are only recommended first line in cases of persistent symptoms of allergic rhinitis where nasal blockage is the primary complaint, ‘or nasal polyps are present. aCe Coca cg Intranasal decongestant is incorrect. Although nasal decongestants are sometime in the management of allergic rhinitis intranasal antihistamines would be first line in this case (see NICE guidelines below), Pecken teen Tag Question ieee Pd kOe Question 19 of 65 A9-year-old child with a history of infantile atopic dermatitis presents complaining of periods of severely itchy eyes, sneezing and a blocked nose. He has noticed this typically occurs on weekends which he spends at his father’s house. On direct questioning he informs you that his father’s house is not as clean as his mother’s and that his father has a number of pets including two dogs and a cat. What is the best first line management plan? Select one answer only Advise the use of house dust mite impermeable covers for pillows and mattresses at his father’s home Arrange skin prick testing Request blood tests for Total and specific IgE levels Start inhaled corticosteroids m)/0/O;o]|> Start oral antihistamine Or un) ‘A9-year-old child with a history of infantile atopic dermatitis presents complaining of periods of severely itchy eyes, sneezing and a blocked nose. He has noticed this typically occurs on weekends which he spends at his father's house. On direct questioning he informs you that his father’s house is not as clean as his mother's and that his father has a number of pets including two dogs and a cat. What is the best first line management plan? Select one answer only Advise the use of house dust mite impermeable covers for pillows and mattresses at his father’s home A Cc D E Request blood tests for Total and specific IgE levels Start inhaled corticosteroids Start oral antihistamine Explanation Allergy testing is indicated when the aetiology is unclear and particularly before arduous allergen avoidance strategies, such as house dust mite avoidance measures, or removal of a pet from the family home are advised. Skin prick testing s the allergy testing of choice. Avoidance of confirmed animal allergens is considered important to control symptoms of allergic rhinitis and reduce the risk of asthma developing. Advise the use of house dust mite impermeable covers for pillows and mattresses at his father's home is incorrect. House dust mite avoidance measures are arduous, expensive and lack a solid evidence base. House dust mite allergy should be confirmed before avoidance measures are recommended. C. Request blood tests for Total and specific IgE levels Allergy testing is indicated when the aetiology is unclear and particularly before arduous allergen avoidance strategies, such as house dust mite avoidance measures, or removal of a pet from the family home are advised. Skin prick testing is the allergy testing of choice. If skin prick testing is not available total and rate IgE levels, RAST (tadioallergosorbent test) or ELISA (enzyme- linked immunosorbent assay) may be useful D. Start inhaled corticosteroids Inhaled corticosteroids are useful however allergen avoidance, if possible, is fundamental in the management of allergic rhinitis so testing for allergens is a better answer. Oral antihistamines are useful however allergen avoidance is fundamental in the management of allergic rhinitis. Rate this question: — Pd kOe Question 20 of 65 A 3-year-old boy is brought to the emergency department with a history of sudden onset noisy breathing. Auscultation reveals significant wheeze. He is initially managed with nebulised bronchodilators with no improvement. What is the most likely diagnosis? Acute Severe Asthma Anaphylaxis C1 esterase deficiency Inhaled foreign body m)/OU/O/;/o/> Viral Induced Wheeze ON are) ‘A 3.year-old boy is brought to the emergency department with a history of sudden onset noisy breathing. Auscultation reveals significant wheeze. He is initially managed with nebulised bronchodilators with no improvement. What is the most likely diagnosis? Acute Severe Asthma Anaphylaxis C1 esterase deficiency Viral Induced Wheeze A B Cc iE Explanation % Foreign body inhalation should always be amongst the differential diagnosis of wheeze in preschool children. The history of sudden onset difficulty in breathing and wheeze is characteristic. On auscultation the wheeze is typically monophonic as the foreign body obstructs ‘one localised area (as opposed to widespread bronchoconstriction in conditions such as viral induced wheeze and asthma that are associated with polyphonic wheeze). Acute Severe Asthma is incorrect. Asthma is frequently associated with an identifiable trigger such as house dustmite, smoke and exercise amongst others. At this age a definite diagnosis of ‘asthma would be unusual as most children prone to wheeze tend to present following viral illnesses with the majority growing out of this over time, the term viral induced wheeze or episodic wheeze is commonplace for preschool wheezers. Although exacerbations of asthma and viral induced wheeze may develop quickly, the very sudden onset of symptoms in this case makes foreign body inhalation more likely. The lack of any improvement with the use of a bronchodilator also supports foreign body inhalation. ‘Anaphylaxis is incorrect. Anaphylaxis would usually present with other features such as urticaria, angioedema, vomiting and cardiovascular compromise. There may be a history of exposure to a known or potential allergen (e.g. nuts, fish etc.) C1 esterase deficiency is incorrect. Hereditary Angioedema is a genetic condition that ocours as a result of low or abnormal C1 esterase inhibitor. Angioedema is the cardinal feature, episodes may. ‘occur spontaneously or secondary to a variety of triggers. The angioedema may affect the skin, ‘gastrointestinal tract or the upper airway. Angioedema of the upper airway will result in stridor as opposed to wheeze which is a lower respiratory tract sign. Viral Induced Wheeze is incorrect. Viral induced wheeze Is associated with a history of coryzal illness, fever and cough. The sudden onset of symptoms makes this less likely that FB inhalation. The lack of any improvement with the use of a bronchodilator also suggests FB inhalation is more likey ne eon _ Pd kOe Question 21 of 65 ‘An 8-year-old child admitted with an acute exacerbation of asthma is ready for discharge from the paediatric ward. He has never required admission to HDU. He has been using his salbutamol inhaler on most days over the past 3 months and frequently wakes with nocturnal cough. His current regular medication consists of beclometasone 100 micrograms twice daily and salbutamol when required. His inhaler technique is good and there are no indications of poor compliance What, if any, changes would you make to his current management? Continue current treatment Add long-acting beta-2 agonist (e.g. salmeterol) Urgent referral to specialist paediatric pulmonologist Increase steroid inhaler to 400 micrograms per day m)/OU/O;o|> Increase steroid inhaler to 800micrograms per day Cr ura) ‘An 8-year-old child admitted with an acute exacerbation of asthma is ready for discharge from the paediatric ward. He has never required admission to HDU. He has been using his salbutamol inhaler on most days over the past 3 months and frequently wakes with nocturnal cough. His current regular medication consists of beclometasone 100 micrograms twice dally and salbutamol when required. His inhaler technique is good and there are no indications of poor compliance. What, if any, changes would you make to his current management? Continue current treatment A c D E Urgent referral to specialist paediatric pulmonologist Increase steroid inhaler to 400 micrograms per day Increase steroid inhaler to 800micrograms per day Explanation This child is currently on step two of the British Thoracic Society and SIGN guidelines for management of Asthma in children aged 5-12 years of age. His asthma remains poorly controlled with nocturnal symptoms and frequent requirement for rescue medication. He requires escalation to step three of which an inhaled long-acting beta-2 agonist (LABA) such as salmeterol is preferable. The response to the LABA should then be reviewed, if there is a response but control Temains inadequate the inhaled corticosteroid dose should be increased but the LABA continued. If there is no response it should be stopped and the inhaled corticosteroid dose should be increased eee Continue current treatment is incorrect. Continue current treatment is not an option given that this child is currently on step two of the British Thoracic Society and SIGN guidelines for management of Asthma. His asthma remains poorly controlled with nocturnal symptoms and frequent requirement for rescue medication. He requires escalation to step three. C. Urgent referral to specialist paediatric pulmonologist Referral to pulmonologist is incorrect. Escalation to step three of the BTS/SIGN treatment ladder is indicated. Inhaled long-acting beta-2 agonist (LABA) such as salmeterol is fist line. A referral will be warranted if additional controller therapies have failed to provide relief in the patient. D. Increase steroid inhaler to 400 micrograms per day Increase steroid inhaler to 400 micrograms per day is incorrect. This is an option as the child is currently on step two and requires escalation to step three however inhaled long-acting beta-2 agonist (LABA) such es salmeterol should be trialled first line. The response to the LABA should then be reviewed, if there is a response but control remains inadequate the inhaled corticosteroid dose should be increased to 400 micrograms per day but the LABA continued. If there is no response it should be stopped and the inhaled corticosteroid dose should be increased to 400 micrograms per day. E, Increase steroid inhaler to 800micrograms per day Increase steroid inhaler to 800 micrograms per day is incorrect, This is step four of the BTS/SIGN treatment ladder. The child is currently on step two and requires escalation to step three. Rate this question: aE ToRe Cll) Feedback OP kl Question 22 of 65 ‘A 4-year-old child is brought in for review following a recent admission to hospital with wheeze. He has a history of atopy with eczema as an infant and allergic rhinitis in the summer months. He is currently on inhaled salbutamol PRN. His mother reports she usually gives him his inhaler on four or five days per week. You discuss possible management options. His mother is strongly opposed to using steroids. What is the next best step in management? Refer to Respiratory Paediatrician Sodium Cromoglycate Long-acting beta-2 agonist (e.g. salmeterol) Leukotriene receptor antagonist (Montelukast) m)/OU)/O/a/]> Oral Theophylline ON arse) ‘A 4-year-old child is brought in for review following a recent admission to hospital with wheeze. He has a history of atopy with eczema as an infant and allergic rhinitis in the summer months. He is currently on inhaled salbutamol PRN. His mother reports she usuelly gives him his inhaler on four or five days per week. You discuss possible management options. His mother Is strongly opposed to using steroids. What is the next best step in management? Refer to Respiratory Paediatrician Sodium Cromoglycate Long-acting beta-2 agonist (e.g. salmeterol) A B c iE Oral Theophylline Explanation i This 4-year-old child is currently on step one of the British Thoracic Society and SIGN guidelines for management of Asthma in children aged less than 5 years. An escalation to step two is indicated, inhaled corticosteroids (200 micrograms per day) would typically be the next drug ‘added in but montelukast, a leukotriene receptor antagonist, should be used where there is reason to refrain from using inhaled corticosteroids. Patient / parent preference should be taken into account when formulating a management plan. In this case the parent's concerns about Using steroids should be addressed however the question is looking for strong candidates to demonstrate their familiarity with the guidelines, Tr nau Refer to Respiratory Paediatrician is incorrect. ‘Refer to respiratory paediatrician’ is incorrect because this 4-year-old child is currently on step one of the British Thoracic Society and SIGN {uidelines for management of Asthma in children aged less than 5 years. A step up the ladder to step two is required, referral to a paediatric respiratory subspecialist is step four of the ladder. Cre ei} ‘Sodium Cromoglycate is incorrect. Sodium Cromoglycate may be considered particularly with ‘exercise induced asthma in older children but its use is controversial PC cu Oc Long-acting beta-2 agonist (e.g. salmeterol) is incorrect. Long-acting beta-2 agonists are part of the treatment in step three of the ladder for children aged 5-12 years, they do not feature in the treatment ladder for children less than § years of age Oral Theophylline is incorrect. Oral Theophylline does not feature in the British Thoracic Society and SIGN guidelines for management of Asthma in children aged less than 5 years. Rate t teen Tag Question ose a Question 23 of 65 You are called to review a 6-year-old child known to have asthma. He has presented with a short history of worsening dyspnoea and cough. His regular medication consists of beclometasone 100 micrograms twice daily and salbutamol when required. He last took his salbutamol inhaler one hour ago. His observations are as follows; heart rate 110, respiratory rate 26, oxygen saturations 96%. On examination he is able to talk in full sentences. Respiratory examination demonstrates good air entry bilaterally with polyphonic wheeze and a prolonged expiratory phase. What is the next best step in management? 10 puffs Salbutamol via spacer 5mg nebulised Salbutamol 2.5 mg nebulised Salbutamol Inhaled Ipratropium m)/OU;/O/;/oO/]> Oral Prednisolone Or UPC) You are called to review a 6-year-old child known to have asthma. He has presented with a short history of worsening dyspnoea and cough. His regular medication consists of beclometasone 100 micrograms twice daily and salbutamol when required. He last took his salbutamol inhaler one hour ago. His observations are as follows; heart rate 110, respiratory rate 26, oxygen saturations 96%. On examination he is able to talk in full sentences, Respiratory examination demonstrates {good air entry bilaterally with polyphonic wheeze and a prolonged expiratory phase. What is the next best step in management? ei 5mg nebulised Salbutamol Inhaled Ipratropium B C | 2.5mgnebulised Salbutamo! D E_ | oral prednisolone Explanation 10 puffs Salbutamol via spacer is the correct answer. Inhaled beta-2 agonists (e.9. salbutamol) are the first line treatment for the management of acute asthma, Delivery via a metered-dose inhaler and spacer is the favoured choice unless there is an oxygen requirement / severe or ife- threatening attack. eee roy ‘5mg nebulised Salbutamol is incorrect. 5 mg nebulised Salbutamol is incorrect because nebulised administration is not required in this case, ene cy 2.5 mg nebulised Salbutamol is incorrect. As above, nebulised administration is not required in this case, If nebulised salbutamol was to be given the dose for children aged > 5 years is 5mg. Inhaled Ipratropium is incorrect. Nebulised ipratropium bromide (250 micrograms for age > 2 years, 125 micrograms <2 years) should be added in severe / rns threatening asthma where beta: 2if there is a poor response to the beta-2 agonist alone. It is not appropriate here. Oral Predhisolone is incorrect. Steroids should be given eerly in the management of acute asthma but inhaled beta-2 agonists (e.9. salbutamol) are the first line treatment. Prednisolone should be used with doses of 20 mg for children aged 2 ~ 5 years and 30-40 mg for those older than 5. It is ‘common for children to vomit after administration of prednisolone, the dose should be repeated if this is the case. IV hydrocortisone is indicated if oral prednisolone is not tolerated. A’ day course of prednisolone is most commonly used but can be extended if symptoms persist. PoLCRUe Del Deena Beet Question 24 of 65 A 4-year-old is being discharged from the paediatric ward following admission for an ear infection complicated by vomiting and poor oral intake. On examination he initially had bilaterally erythematous and bulging tympanic membranes. This has now settled but evidence of middle ear effusions are visible on otoscopy. His parents are concerned about how this will affect his hearing What would be the best advice to give his parents? He is at risk of developing speech delay He is likely to have hearing impairment in the short term He is likely to have permanent hearing imparment He will require antibiotics m)/OU)/O/a/]> He will require a cochlear implant Or ure) ‘A 4-year-old is being discharged from the paediatric ward following admission for an ear infection complicated by vomiting and poor oral intake. On examination he initially had bilaterally erythematous and bulging tympanic membranes. This has now settled but evidence of middle ear effusions are visible on otoscopy. His parents are concerned about how this will affect his hearing What would be the best advice to give his parents? A Cc D IE He is at risk of developing speech delay He is likely to have permanent hearing imparment He will require antibiotics He will require a cochlear implant Explanation The scenario describes a case of otitis media with effusion which is the most common cause of hearing impairment in children typically affecting those of pre-school age. The condition usually resolves spontaneously therefore no active intervention is required initially and a period of watchful waiting is advocated. ee ny He is at risk of developing speech delay is incorrect. Otitis media with effusion can result in speech delay if severe and prolonged however in most cases the hearing impairment is short lived. The child in the question is 4 years of age, hearing impairment in younger children with repidly developing speech and language are most at risk. Oe eg ie eu cea He is unlikely to have permanent hearing impairment is incorrect. Children who develop middle ear effusions, often with a history of recurrent ear infections, commonly present with hearing impairment however this is typically short term hearing loss. A period of watchful waiting is therefore usually adopted initially. De ed He will require antibiotics is incorrect. Otitis media with effusion usually resolves spontaneously therefore no active intervention is required initially and a period of watchful waiting is advocated. Antibiotics should not be prescribed so stem D is incorrect. ae ee eee He will require a cochlear implant is incorrect. As most cases of otitis media with effusion improve, watchful waiting is typicaly the initial management of choice. Children with significant hearing impairment or ongoing symptoms require referral for assessment by an ENT specialist. The hearing impairment in otitis media with effusion is conductive, surgical treatment consists of insertion of grommets to allow drainage from the middle ear. Cochlear implants may be used for children with sensorineural deafness which is severe or profound. Cee ono eae Question 25 of 65 You are preparing to discharge a 14-year-old boy home following a month long admission for bacterial meningitis. You are asked to arrange the necessary audiology follow up. What type of hearing impairment may be expected following bacterial meningitis? Combined conductive and sensorineural deafness Conductive deafness Isolated Neural deafness No hearing impairment Sensorineural deafness m/oU/O;|o|> Or uray You are preparing to discharge a 14-year-old boy home following a month long admission for bacterial meningitis. You are asked to arrange the necessary audiology follow up. What type of hearing impairment may be expected following bacterial meningitis? Combined conductive and sensorineural deafness Conductive deafness Isolated Neural deafness A B Cc D Explanation No hearing impairment m Bacterial meningitis may result in a sensorineural hearing impairment which can be bilateral and severe to profound. Hearing impairment is in fact the most common long-standing complication ‘of bacteriel meningitis with 10% of survivors of the iiness developing some degree of hearing loss. The infection and secondary inflammatory process may damage the cochlea and the auditory nerve. ‘A. Combined conductive and sensorineural deafni Combined conductive and sensorineural deafness is incorrect. The hearing loss following bacterial meningitis is purely sensorineural not conductive, The infection and secondary inflammatory process may damage the cochlea and the auditory nerve but the middle and outer fear are spared. Accordingly stem A is incorrect. B. Conductive deafness Conductive deafness is incorrect. The hearing loss following bacterial meningitis is sensorineural not conductive, aren eR Isolated Neural deafness is incorrect. Isolated neural deafness refers to pathology purely of the auditory nerve. Although bacterial meningitis may damage the auditory nerve a number of studies have localised the cause of the hearing loss to the cochlea making ‘sensorineural deafness’ the best answer to this question. No hearing impairment is incorrect. Hearing impairment is in fact the most commen long- standing complication of bacterial meningitis with 10% of survivors of the illness developing some degree of hearing loss. Chen teeta Tag Question iene Question 26 of 65 A 6-year-old child is referred by his school with concerns that he has difficulty hearing the teacher's instructions in class. Which of the following hearing tests is most appropriate to assess his hearing? Behavioural observation audiometry (BOA) Distraction testing Play audiometry Pure tone audiometry m/oU/O;|o|> Visual reinforcement audiometry (VRA) Or ure) ‘A 6-year-old child is referred by his school with concerns that he has difficulty hearing the teacher's instructions in class. Which of the following hearing tests is most appropriate to assess his hearing? Behavioural observation audiometry (BOA) Distraction testing Play audiometry A B c =| E Explanation Visual reinforcement audiometry (VRA) Pure tone audiometry is the audiclogical test of choice for older children and adults. It may be used for children aged 5 years and older, although this obviously depends upon the individual child BOA is appropriate for infants up to six months of age. A stimulus is presented to the infant via a speaker. The examiner looks for any change in behaviour suggesting the sound has been heard. Distraction testing is used for children aged between the age of 7 and 30 months. The examiner makes sounds behind the infant/child ensuring the object producing the sound is out of view. The response to the sound is observed. Ad Play audiometry may be used for children between 30 months and § years of age. Games are played that require the child to respond to a sound stimulus eg the child holds a ball, when a sound his heard they are required to place the ballin a container. E, Visual reinforcement audiometry (VRA) \VRA is appropriate for those aged between 6 and 30 months. When the child responds to the sound stimulus they are rewarded visually with a toy that illuminates/ moves to reinforce their listening behaviour, ECR UTN WsiCoLF aE TRen CCl) Feedback Question 27 of 65 A 7-year-old was recently discharged from the paediatric ward following admission for an ear infection complicated by poor oral intake and vomiting. At initial presentation examination showed bilaterally erythematous and bulging tympanic membranes. His parents are concerned that he is struggling to hear them speak at times. They ask you to have a look at his ears. On otoscopy you find evidence of an effusion on the left side. You proceed to perform Rinne’s and Weber's tests. Which of the following would support your diagnosis of a left-sided middle ear effusion? Weber's localising to left, Rinne negative on left, Rinne positive on right Weber's localising to left, Rinne positive on left, Rinne negative on right Weber's localising to right, Rinne negative on left, Rinne positive on right Weber's localising to right, Rinne positive on left, Rinne negative on tight Weber's localising to left, Rinne positive on left, Rinne positive on tight m|;/oO|/O|]owa|> Ore Pe) ‘A 7-year-old was recently discharged from the paediatric ward following admission for an ear infection complicated by poor oral intake and vomiting. At initial presentation examination showed bilaterally erythematous and bulging tympanic membranes. His parents are concerned that he is struggling to hear them speak at times. They ask you to have a look at his ears. On otoscopy you find evidence of an effusion on the left side. You proceed to perform Rinne's and Weber's tests. Which of the following would support your diagnosis of a left-sided middle ear effusion? Weber's localising to left, Rinne positive on left, Rinne negative on B right | Webers localising to right, Rinne negative on left, Rinne positive on right D Weber's localising to right, Rinne positive on left, Rinne negative on right E Weber's localising to left, Rinne positive on left, Rinne positive on right Explanation x Option A is correct. Weber's test involves the placement of a 512 Hz tuning fork on the centre of the child's forehead. The child is then asked where they can hear the sound the loudest. The normal response is for it to be heard in the middle ie. not louder on any one side. Ifthe child reports hearing the sound louder in one ear than the other this may mean one of two things; either there is conductive hearing impairment on that side or there is sensorineural deafness on the side in which the sound is heard less. Rinne’s testis then required to distinguish between the two. Rinne's test is performed by placing the 512 Hz tuning fork in two positions and asking the child to inform you which is louder. Position 1 is with the tuning fork placed on the mastoid process (vibration conducts through the skull and therefore bypasses the outer ear), position two is with the tuning fork held adjacent to the pinna (Which relies on the outer ear to transmit the sound to the middle and inner ear). In those without any hearing impairment position 2 (alr conduction) should be louder than position’ (bone conduction). This normal response is referred to as a positive result ie. Rinne positive In this question the stem informs the candidate of a left sided effusion (otitis media with effusion) therefore one should deduce the Weber and Rinne tests should support a left sided conductive deafness. Weber's would therefore lateralise to the left (heard loudest on the left). Rinne’s on the affected left side would reveal the patient hears the sound loudest on the mastoid process as this bypasses the conductive mechanisms of the outer and middle ear of which function is impaired secondary to fluid in the middle ear. This abnormal result, where bone ‘conduction is louder than air conduction, is referred to as Rinne negative. The stem does not suggest any problem with the right ear and therefore a normal Rinne positive result is expected. B. Weber's localising to left, Rinne positive on left, Rinne negative on right Weber's localising to left, Rinne positive on left, Rinne negative on right ~ these results would support a diagnosis of a combined conductive and sensorineural hearing impairment on the right side making this answer incorrect. Fe ou LCC Weber's localising to right, Rinne negative on left, Rinne positive on right ~ these results would support a diagnosis of a combined conductive and sensorineural hearing impairment on the left side making this answer incorrect. D. Weber's localising to right, Rinne positive on left, Rinne negative on right Weber's localising to right, Rinne positive on left, Rinne negative on right ~ these results would support a diagnosis of a right sided conductive deafness making this answer incorrect. CN eee Ra a Sed Weber's localising to left, Rinne positive on left, Rinne positive on right ~ these results would support a diagnosis of a sensorineural hearing impairment on the right side making this answer incorrect, Berets ese rT = Py Question 28 of 65 15-year-old presents complaining of having difficulty hearing on his left side. He recently had a significant hospital admission for suspected meningitis although investigations were negative: blood culture, CSF culture and pneumococcal & meningococcal PCR. You examine his external auditory canal with an otoscope which shows bilaterally pink tympanic membranes with no sign of effusion. You proceed to perform Weber's and Rinne's tests. Which of the following would support the most likely cause of his hearing impairment? Weber's localising to left, Rinne negative on left, Rinne positive on right Weber's localising to left, Rinne positive on left, Rinne negative on right Weber's localising to right, Rinne negative on left, Rinne positive on right Weber's localising to right, Rinne positive on left, Rinne positive on right m|/oU|/O/;W]> Weber's localising to left, Rinne positive on left, Rinne positive on tight ON ure) ‘A 15-year-old presents complaining of having difficulty hearing on his left side. He recently had a significant hospital admission for suspected meningitis although investigations were negative: blood culture, CSF culture and pneumococcal & meningococcal PCR. You examine his external auditory canal with an otoscope which shows bilaterally pink tympanic membranes with no sign of effusion. You proceed to perform Weber's and Rinne's tests. Which of the following would support the most likely cause of his hearing impairment? Weber's localising to left, Rinne negative on left, Rinne positive on right Weber's localising to left, Rinne positive on left, Rinne negative on right Weber's localising to right, Rinne negative on left, Rinne positive on right A B Cc E Weber's localising to left, Rinne positive on left, Rinne positive on right Explanation Dae Option D is correct. This question requires the knowledge that sensorineural hearing impairment is a complication of bacterial meningitis. it then tests your understanding of the Weber and Rinne screening tests and how they have some ability to differentiate conductive and sensorineural hearing impairment at the bedside. Weber's test involves the placement of a 12 Hz tuning fork on the centre of the child's forehead The child is then asked where they con hear the sound the loudest. The normal response is for it to be heard in the middle ie. not louder on any one side. I the child reports hearing the sound louder in one ear than the other this may mean one of two things; either there is conductive hearing impairment on that side or there is sensorineural deafness on the side in which the sound is heard less. Rinne's testis then required to distinguish between the two. Rinne's test is performed by placing the 512 Hz tuning fork in two positions and asking the child to inform you which is louder. Position 1 is with the tuning fork placed on the mastoid process (vibration conducts through the skull and therefore bypasses the outer ear), position two is with the tuning fork held adjacent to the pinna (which relies on the outer ear to transmit the sound to the middie and inner ear). In those without any hearing impairment position 2 (air conduction) should be louder than position’ (bone conduction). This normal response is referred to as a positive result ie. Rinne positive, Bacterial meningitis is associated with sensoimeural hearing loss. The question informs the candidate that the hearing impairment is on the left in this case. It should therefore be deduced that the Weber and Rinne's test results should support the diagnosis of a left sided sensorineural deafness. Weber's localising to the right may mean either conductive deafness on that side or sensorineural deafness on the opposite side. Moving on to Rinne's test next and finding a positive (normal) Rinne’s test on the right there must be sensorineural deafness on the left. Finally a positive (normal) Rinne's test on the left suggests an isolated sensorineural deafness on the left. (Nb. if the Rinne’s test was abnormal on the left this, would suggest combined conductive and sensorineural deafness on the left) Le ee ee a Ru Cu aed Weber's localising to left, Rinne negative on left, Rinne positive on right - these findings should identify a left-sided sensorineural deafness. This answer is incorrect as the results described suggest a left sided conductive hearing loss. B. Weber's localising to left, Rinne positive on left, Rinne negative on right Weber's localising to left, Rinne positive on left, Rinne negative on right - these findings should identify a left-sided sensorineural deafness. This answer is incorrect as the results described suggest a combined conductive and sensorineural hearing loss on the right. Weber's localising to right, Rinne negative on left, Rinne positive on right Weber's localising to right, Rinne negative on left, Rinne positive on right -these findings should identify a left-sided sensorineural deafness. This answer is incorrect as the results described ‘suggest a combined conductive and sensorineural hearing loss on the left. E, Weber's localising to left, Rinne positive on left, Rinne positive on right Weber's localising to left, Rinne positive on left, Rinne positive on right - this answer is incorrect as the results described suggest a sensorineural hearing impairment on the right side whereas the stem reports hearing impairment on the left. ate this question Question 29 of 65 A 14-year-old girl is admitted with a 5-day history of coryza, headache and non-productive cough; she has been given amoxicillin by her GP but has failed to improve. On examination she is pyrexial with a temperature of 37.7°C. She is mildly tachypnoeic and requiring wafting oxygen to maintain her saturations at > 95%. On auscultation there is good air entry bilaterally along with scattered wheeze. She has no PMH of Asthma Which organism is most likely to be responsible for her clinical condition? Bordetella Pertussis Haemophilus Influenzae Mycoplasma Pneumoniae Staphylococcus Aureus m)/OU)/O/a/]> Streptococcus Pneumoniae Cree uray A 14-year-old girl is admitted with a 5-day history of coryza, headache and non-productive cough; she has been given amoxicillin by her GP but has failed to improve. On examination she is pyrexial with a temperature of 37.7°C. She is mildly tachypnoeic and requiring wafting ‘oxygen to maintain her saturations at > 95%. On auscultation there is good air entry bilaterally along with scattered wheeze. She has no PMH of Asthma. Which organism is most likely to be responsible for her clinical condition? Bordetella Pertussis Haemophilus Influenzae lo) A B Staphylococcus Aureus D Ir Streptococcus Pneumoniae Explanation This is an atypical bacterium and one of the commonest cause of community acquired pneumonia (identified in approximately 30% of children admitted to hospital). Historically it's thought to affect school aged children and adolescents although recent studies suggest it is not also unusual in young children under five years of age. Ahistory of headache or sore throat should prompt one to consider a viral or atypical aetiology. ‘Auscultatory findings of widespread wheeze with the absence of focal signs would also support a viral or atypical cause. In this case the child has also had a poor response to first line antibiotics, further supporting a possible viral or atypical organism. Pneumonia secondary to Mycoplasma Pneumoniae is usually mild but can be associated with complications in a number of systems including the skin (erythema multiforme, erythema nodusum and generalised macularpapular rash) and nervous system (meningitis/encephalitis and Guillain-Barre Syndrome) Chlamydia Pneumoniae is a further atypical bacterium known to be a common cause of ‘community acquired pneumonia. British Thoracic Society guidelines for the management of community acquired pneumonia advocate the use of a macrolide antibiotic if Mycoplasma or Chlamydia infection is suspected on clinical grounds or if there is poor response to first line treatment. Itis worth noting that the causative organism in community acquired pneumonia frequently remains ut lentified, particularly in young children who are unable to expectorate sputum, ee a a oy Bordetella Pertussis is incorrect. This is the organism responsible for whooping cough. This is most commonly seen in young infants. Vaccination is offered to pregnant mothers and is part of the routine childhood immunisation schedule, first given at 8 weeks of age. Cue ad Haemophilus influenzae is incorrect. This is a significant cause of pneumonia worldwide but infrequentty seen within the UK since the introduction of the Hib vaccine DT eee Ud ‘Staphylococcus Aureus is incorrect. This is a rare cause within the UK (may be seen in infants or {as a secondary infection in young children with influenza). Pneumonia as a result of staphylococcus aureus infection is classically associated with ‘ust’ coloured sputum in those old ‘enough to expectorate it. There is a higher association with serious illness and complications, such as empyema, In this case however the examination findings of wheeze in an adolescent, on the background of poor response to frst line antibiotics and symptoms such as headache / sore throat should prompt consideration of viral or atypical organisms So eres aU ‘Streptococcus Pneumoniae is incorrect. This is the most common bacterial cause of community ‘acquired pneumonia in children. In this case however the examination findings of wheeze in an adolescent, on the background of poor response to first line antibiotics and symptoms such as headache / sore throat should prompt consideration of viral or atypical organisms, LZ -Cle] ello Question 30 of 65 A 5-year-old is referred for assessment by the GP with a history of fever and increased work of breathing. His mother reports that he has been generally unwell and felt ‘hot’ for the past 48 hours. He is eating and drinking as usual. His immunisations are up-to-date. Observations show a fever of 38.8°C, heart rate of 120, respiratory rate of 32 and oxygen saturations of 98%. On examination he appears fairly well although coughing intermittently. There is mild increased work of breathing with subtle subcostal recession. On auscultation you note crepitations at the left base. What is the best next step in the management in this child? Admit for lV Co-amoxiclav Check inflammatory markers (CRP, FBC) and perform a chest X-ray Check inflammatory markers (CRP, FBC), take a blood culture and perform a chest X-ray Perform chest X-ray and if it confirms pneumonia, prescribe oral Amoxicillin and discharge with advice m)/OU/O;]o) > Prescribe oral Amoxicillin and discharge with advice Or UR) ‘A 5:year-old is referred for assessment by the GP with a history of fever and increased work of breathing. His mother reports that he has been generally unwell and felt hot' for the past 48 hours. He is eating and drinking as usual. His immunisations are up-to-date. Observations show a fever of 38.8°C, heart rate of 120, respiratory rate of 32 and oxygen saturations of 98%. On examination he appears fairly well although coughing intermittently. There is mild increased work of breathing with subtle subcostal recession. On auscultation you note crepitations at the left base. What is the best next step in the management in this child? Correct Admit for lV Co-amoxiclav Check inflammatory markers (CRP, FBC) and perform a chest X-ray perform a chest X-ray Perform chest X-ray and if it confirms pneumonia, prescribe oral Amoxicillin and discharge with advice Explanation x A B | Gheck inflammatory markers (CRP, FBC), take a blood culture and D m Prescribe oral Amoxicilin and discharge with advice - This child has community acquired pneumonia, Pneumonia may be viral or bacterial, distinguishing the two is often difficult. Consider bacterial cause in a child with tachypnoea and signs of respiratory distress along with a persistent fever more than 38.5°C. Chest X-ray and blood tests are not routinely indicated and do not reliably indicate the aetiology. Oral Amoxicillin is the first line treatment for community ‘acquired bacterial pneumonia in children. This child is generally well and should therefore be managed at home. Advice should be given to his mother regarding preventing dehydration and the appropriate use of antipyretics for fever associated with discomfort. Provide a safety net by highlighting signs of deterioration and who to contact if any concerns arise a ee Wag Admit for IV Co-amoxiclav is incorrect. This child has community acquired pneumonia for which ‘oral Amoxicillin is the first line treatment, He is generally well and should therefore me managed at home. B. Check inflammatory markers (CRP, FBC) and perform a chest X-ray Check inflammatory markers (CRP, FBC) and perform a chest X-ray is incorrect. This child has ‘community acquired pneumonia which is a clinical diagnosis, chest Xray and blood tests are not, routinely indicated. C. Check inflammatory markers (CRP, FBC), take a blood culture and perform a chest X-ray Check inflammatory markers (CRP, FBC), take a blood culture and perform a chest X-ay is incorrect. This child has Sena ee pneumonia which is a clinical diagnosis, chest X-ray ‘and blood tests are not routinely indicated. A blood culture is not indicated unless there are concerns that the child is septic, which in this scenario is not the case. form chest X-ray and if it confirms pneumonia, prescribe oral Amoxicillin and Perform chest Xay and if it confirms pneumonia prescribe oral Amoxicillin and discharge with ‘advice is incorrect. This child has community acquired pneumonia which is a clinical diagnosis, chest X-ray is not routinely indicated _ Nay CeO Question 31 of 65 A 5-year-old is referred for assessment by the GP with a history of fever and increased work of breathing. He initially presented to the GP 4 days ago when is mother reported that he had been ‘generally unwell and felt hot’ for the previous 48 hours. At that time he was noted to be tachypnoeic and had some crepitations at his left base. He was otherwise well and was treated in the community for a lower respiratory tract infection with oral amoxicillin. His mother brought him back to the surgery as he had not improved and continued to ‘have a temperature’. He has been drinking but not as much as usual. Observations show a fever of 38.8°C, a heart rate of 150, respiratory rate of 40 and oxygen saturations of 97%. On examination he appears miserable but alert and interactive. He is tachypnoeic and has mild recession. On auscultation you note reduced breath sounds at the left base with a dull percussion note. What is the best next step in the management if this child? Admit, check inflammatory markers (CRP, FBC), prescribe IV Coamoxiclav Admit for lV Co-amoxiclav Admit, perform a chest X-ray, prescribe lV Coamoxiclav Perform chest X-ray and if confirms pneumonia prescribe oral Amoxicillin and discharge with advice m/O0/O/;B]|> Prescribe oral Amoxicillin and discharge with advice Or ui) ‘A S-year-old is referred for assessment by the GP with a history of fever and increased work of breathing, He initially presented to the GP 4 days ago when is mother reported that he had been ‘generally unwell and felt hot' for the previous 48 hours. At that time he was noted to be tachypnoeic and had some crepitations at his left base. He was otherwise well and was treated in the community for a lower respiratory tract infection with oral amoxicillin, His mother brought him back to the surgery as he had not improved and continued to ‘have a temperature’. He has been drinking but not as much as usual. Observations show a fever of 38,8°C, a heart rate of 150, respiratory rate of 40 and oxygen saturations of 97%. On ‘examination he appears miserable but alert and interactive. He is tachypnoeic and has mild recession. On auscultation you note reduced breath sounds at the left base with a dull percussion note. What is the best next step in the management if this child? ‘Admit, check inflammatory markers (CRP, FBC), prescribe IV Coamoxiclav Admit for IV Co-amoxiclav oO A B cl D_ | Perform chest x-ray and if confirms pneumonia prescribe oral Amoxicillin and discharge with advice Prescribe oral Amoxicillin and discharge with advice Explanation * ‘Admit, perform a chest X-ay, prescribe IV Coamoxiclav ~ This child has developed an empyema This complication of pneumonia should always be considered if fever is persistent for seven days or continues despite 48 hours of appropriate antibiotics. The chest X-ray may demonstrate the presence of fluid in the pleural space but an ultrasound will be required to confirm the diagnosis and estimate the volume of fluid. Intravenous antibiotics are always indicated (follow local microbiology guidelines). If drainage is required this is typically undertaken in a tertiary centre. Children who have pneumonia complicated by empyema should be followed up after discharge to ‘ensure complete recovery and resolution of radiographic changes. ‘A. Admit, check inflammatory markers (CRP, FBC), prescribe IV Coamoxiclav ‘Admit, check inflammatory markers (CRP, FBC), prescribe IV Coamoxiclav is incorrect. This child has developed an empyema. The examination findings of reduced air entry and dull percussion note may be as a result of extensive consolidation, a parapneumonic effusion or empyema. Plain chest X-ray may help to identify these complications whereas blood tests will not. Blood tests are Useful however to look for other complications of pneumonia such as anaemia and SIADH Inflammatory markers, such as CRP, may be used to monitor disease progession. CeCe aNy oS ‘Admit for IV Co-amoxiclav is incorrect. The child has likely developed an empyema. All children with pneumonia complicated by empyema should be admitted to hospital and must indeed receive intravenous antibiotics. Imaging is required to confirm the diagnosis. D. Perform chest X-ray and if confirms pneumonia prescribe oral Amoxicillin and discharge Reco Perform chest X-ray and if confirms pneumonia prescribe oral Amoxicilin and discharge with advice is incorrect. In view of the examination findings of reduced air entry and duliness to percussion on the background of persistent fever a chest X-ray is indeed indicated to look for ‘evidence of a parapneumonic effusion. The child has likely developed an empyema. All children with pneumonia complicated by empyema should be admitted to hospital and must receive intravenous antibiotics. eee EUS CRE Prescribe oral Amoxicillin and discharge with advice is incorrect. This child has been febrile for six days and not responded to 48 hours of firs line antibiotics for community acquired pneumonia, There are examination findings that may suggest a pleural effusion. The child has developed an empyema and requires admission for intravenous antibiotics and possible drainage of the collection. Tag Question ios o 01:29 6 Question 32 of 65 A 4-month-old infant is admitted via the emergency department with a history of ‘turning blue’ following coughing fits. The child has been coryzal for a few days, but has today had protracted episodes of coughing, Clinical examination was unremarkable and he is observed on the paediatric ward. You are called by the nursing staff to review the infant urgently after they witnessed him become 'cyanosed briefly’ after a coughing fit. Examination is once again unremarkable. Which of the following investigations is most likely to help confirm the underlying diagnosis? Blood culture Chest X-ray Echocardiography FBC and blood film m)/OU/O;o|> Nasopharyngeal swab Or unre) ‘A 4-month-old infant is admitted via the emergency department with a history of ‘turning blue! following coughing fits. The child has been coryzal for a few days, but has today had protracted episodes of coughing, Clinical examination was unremarkable and he is observed on the paediatric ward. You are called by the nursing staff to review the infant urgently after they witnessed him become ‘cyanosed briefly’ after a coughing fit. Examination is once again unremarkable. Which of the following investigations is most likely to help confirm the undertying diagnosis? Blood culture Chest X-ray Echocardiography A B c D FBC and blood film BE vere va Explanation NS Rr This infant has whooping cough (Bordetella Pertussis). Young infants are at particular risk from the infection and may present with apnoeas. The characteristic inspiratory ‘whoop’ is seldom seen in this age group. Diagnosis of Bordetella Pertussis is traditionally by isolation of the ‘organism from a nasopharyngeal / pernasal swab. This method is not perfect however and more sensitive tests (such as PCR) are available in some laboratories. A positive culture from nasopharyngeal swab confirms the diagnosis but itis important to note a negative test does not ‘exclude it. Successful culture becomes less likely further into the illness, itis unlikely to be positive more than two weeks post symptom onset. Treatment of pertussis is with a macrolide antibiotic such as clarithromycin or azithromycin (these have superseded the use of erythromycin). Chemoprophylaxis for contacts is required in some circumstances. Ewer Rent Blood culture is incorrect, Blood culture would be an appropriate investigation to look for sepsis in an unwell infant but based upon the clinical history, which is suggestive of pertussis, it would not ‘confirm the suspected underlying diagnosis. Chest X~ay is incorrect. Chest X-ray would be a useful investigation to look for any signs of lower respiratory tract infection and to ensure a normal cardiac/mediastinal silhouette. It would not however confirm the clinical diagnosis of whooping cough (Bordetella Pertussis). Cae Echocardiography is incorrect. Cardiac conditions must always be amongst the differential diagnosis of infants with episodes of cyanosis. In this case however there is a clear history of ‘coughing episode prior to the episodes. Reena) FBC and blood film is incorrect. This infant has whooping cough (Bordetella Pertussis). This is associated with a lymphocytosis on the full blood count but this does not confirm the diagnosis. Rate this questi _ Oey Ts Question 33 of 65 A 6-month-old infant presents with a 3-day history of poor feeding and cough. Her immunisations are up to date. On examination she is alert and appears comfortable at rest but with mild respiratory distress. She is pyrexial with a temperature of 37.8°C. Auscultation reveals good air entry but bilateral crepitations and wheeze. What is the most likely pathogen responsible for this condition? Human Metapneumovirus Influenza A Parainfluenza Respiratory Syncitial Virus m)/OU;/O;|D|> Steptococcus Pneumoniae Or US) ‘A 6-month-old infant presents with a 3-day history of poor feeding and cough. Her immunisations are up to date. On examination she is alert and appears comfortable at rest but with mild respiratory distress. She is pyrexial with a temperature of 37.8°C. Auscultation reveals good air entry but bilateral crepitations and wheeze. What is the most likely pathogen responsible for this condition? Human Metapneumovirus Influenza A Parainfluenza A B Cc E Explanation ‘Steptococcus Pneumoniae Respiratory Syncttal Virus - Ths infant has bronchiolitis. This is an acute vial respiratory tract infection that affects infants, most commonly between the ages of three to six months. The typical history is of coryzal iliness followed by reduced feeding, cough and respiratory distress Widespread wheeze and crepitations may be audible on auscultation, Respiratory Syncitial Virus (RSV) is the predominant responsible organism. Uc acs Human Metapneumovirus is incorrect. Human metapneumovirus is a cause of bronchiolitis, however RSV is the most common cause. Influenza A is incorrect. Influenza A and B may be responsible for bronchiolitis however RSV is the most common cause Parainfluenza is incorrect. Parainfluenza is a cause of bronchiolitis however RSV is the most ‘common cause. Parainfluenza is a significant cause of viral croup. reer Steptococcus Pneumoniae is incorrect. Streptococcus Pneumoniae is a significant cause of bacterial infections, itis the most common bacterial cause of community acquired pneumonia in children, Bronchiolitis is of viral origin, most commonly RSV. PCa tect Tag Question ee Oey Ts Question 34 of 65 A 6-year-old presents with parental concern relating to persistent coughing. The parents teport she has always been prone to chest infections but they usually clear up with a course of antibiotics. Over the past few weeks she has been coughing up green sputum which has caused alarm. She is fully immunised and review of previous presentations reveals multiple episodes of respiratory illnesses along with complaints of frontal headaches. On examination the child appears small for her age and has palpable fluctuance to her nailbeds. On palpation the apex beat is difficult to locate. Auscultation reveals bilateral coarse crepitations. What is the single best investigation to confirm the clinical diagnosis? Bronchoscopy High resolution CT Lung function tests Sweat test m)/O0/O/;o]|> Total Immuglobulins Or uC) ‘A 6-year-old presents with parental concern relating to persistent coughing. The parents report she has always been prone to chest infections but they usually clear up with a course of antibiotics. Over the past few weeks she has been coughing up green sputum which has caused alarm. She is fully immunised and review of previous presentations reveals multiple episodes of respiratory illnesses along with complaints of frontal headaches. On examination the child appears small for her age and has palpable fluctuance to her nailbeds. On palpation the apex beat is dificult to locate. Auscultation reveals bilateral coarse crepitations. What is the single best investigation to confirm the clinical diagnosis? Bronchoscopy Lung function tests ‘Sweat test A c D E Total Immuglobulins Explanation The history of chronic cough with coarse crepitations and clubbing (Stage 1 clubbing: fluctuant nail bed) points to a diagnosis of bronchiectasis. All of the possible answers may be appropriate for a bronchiectasis work up as cystic fibrosis and immunodeficiency are amongst the differential diagnosis. The diagnosis in this case however is Kartagener’s Syndrome, a subset Primary Ciliary Dyskinesia. The condition is characterised by bronchiectasis, chronic sinusitis and situs inversus. The clues in the stem supporting this diagnosis are frontal headaches suggesting sinusitis and the difficult to locate apex beat which is difficult to palpate as there is dextrocardia with the apex beat located on the right side of the chest. High resolution CT will demonstrate bronchiectasis and situs inversus supporting the diagnosis. The gold standard investigation for diagnosis of Primary Ciliary Dyskinesia is examination of cilia with electron microscopy which requires biopsy of the nasal mucosa Bronchoscopy is incorrect. Bronchoscopy may be useful in cases of localised bronchiectasis to tule out an infialed foreign body or malignancy. Itis not required to obtain a tissue biopsy of ciliated cells for examination under electron microscope as this can be achieved from the nasal mucosa, C. Lung function tests Lung function tests is incorrect. Pulmonary function tests demonstrate an obstructive pattern in primary ciliary dyskinesia but this is non-specific. ‘Sweat test is incorrect. This is the gold standard investigation for the diagnosis of Cystic Fibrosis which would fit with clubbing and chronic cough however the dextrocardia, suggested by the difficulty locating the apex beat, and the frontal headaches / sinusitis point to a diagnosis of Kartagener’s syndrome E. Total immuglobulins Total Immunoglobulins is incorrect. Immunodeficiency is amongst the differential diagnosis for bronchiectasis however the dextrocardia, suggested by the difficulty locating the apex beat, and. the frontal headaches / sinusitis point to a diagnosis of Kartagener's syndrome. aC roRe lel} tet Teg

You might also like